Percent change algebra problem

This topic has expert replies

Answer

A
0
No votes
B
0
No votes
C
4
80%
D
0
No votes
E
1
20%
 
Total votes: 5

User avatar
Senior | Next Rank: 100 Posts
Posts: 52
Joined: Mon Apr 19, 2010 2:45 pm
Location: Los Angeles, CA
Thanked: 6 times
GMAT Score:710

Percent change algebra problem

by TheGmatTutor » Sun Jul 12, 2015 1:58 pm
During the first quarter of the year, a stock increased by x percent. During the second quarter, the stock decreased by y percent. If the stock began the year with a price of s, which of the following represents the price per share at the end of the second quarter, in terms of s, x, and y?

(A) s(1+x)(1-y)
(B) s(x/100)(y/100)
(C) s(1+[x/100])(1-[y/100])
(D) 100s(x+y)
(E) xy(100+[s/100])
If this post was helpful, please click the "Thanks" button.

User avatar
MBA Admissions Consultant
Posts: 2279
Joined: Fri Nov 11, 2011 7:51 am
Location: New York
Thanked: 660 times
Followed by:266 members
GMAT Score:770

by Jim@StratusPrep » Mon Jul 13, 2015 3:46 am
The answer is C. A simple percent increase and percent decrease question. 1 ± percent/100 is all you need.
GMAT Answers provides a world class adaptive learning platform.
-- Push button course navigation to simplify planning
-- Daily assignments to fit your exam timeline
-- Organized review that is tailored based on your abiility
-- 1,000s of unique GMAT questions
-- 100s of handwritten 'digital flip books' for OG questions
-- 100% Free Trial and less than $20 per month after.
-- Free GMAT Quantitative Review

Image

User avatar
GMAT Instructor
Posts: 15539
Joined: Tue May 25, 2010 12:04 pm
Location: New York, NY
Thanked: 13060 times
Followed by:1906 members
GMAT Score:790

by GMATGuruNY » Mon Jul 13, 2015 6:47 am
TheGmatTutor wrote:During the first quarter of the year, a stock increased by x percent. During the second quarter, the stock decreased by y percent. If the stock began the year with a price of s, which of the following represents the price per share at the end of the second quarter, in terms of s, x, and y?

(A) s(1+x)(1-y)
(B) s(x/100)(y/100)
(C) s(1+[x/100])(1-[y/100])
(D) 100s(x+y)
(E) xy(100+[s/100])
Let y=100, implying that the stock price decreases by 100% in the second quarter.
Since the stock loses 100% of its value, the resulting price = $0.
Implication:
When y=100 is plugged into the correct answer choice, a price of $0 must be yielded.
Only C includes a factor that will be equal to 0 when y=100:
(1-[y/100]) = (1- [100/100]) = (1 - 1) = 0.

The correct answer is C.
Private tutor exclusively for the GMAT and GRE, with over 20 years of experience.
Followed here and elsewhere by over 1900 test-takers.
I have worked with students based in the US, Australia, Taiwan, China, Tajikistan, Kuwait, Saudi Arabia -- a long list of countries.
My students have been admitted to HBS, CBS, Tuck, Yale, Stern, Fuqua -- a long list of top programs.

As a tutor, I don't simply teach you how I would approach problems.
I unlock the best way for YOU to solve problems.

For more information, please email me (Mitch Hunt) at [email protected].
Student Review #1
Student Review #2
Student Review #3

GMAT/MBA Expert

User avatar
GMAT Instructor
Posts: 16207
Joined: Mon Dec 08, 2008 6:26 pm
Location: Vancouver, BC
Thanked: 5254 times
Followed by:1268 members
GMAT Score:770

by Brent@GMATPrepNow » Mon Jul 13, 2015 6:47 am
TheGmatTutor wrote:During the first quarter of the year, a stock increased by x percent. During the second quarter, the stock decreased by y percent. If the stock began the year with a price of s, which of the following represents the price per share at the end of the second quarter, in terms of s, x, and y?

(A) s(1+x)(1-y)
(B) s(x/100)(y/100)
(C) s(1+[x/100])(1-[y/100])
(D) 100s(x+y)
(E) xy(100+[s/100])
We can also use the INPUT-OUTPUT approach.
I might take a risk and try some EXTREME VALUES to see if I can eliminate some answer choices.
If x and y both equal zero, then the value of the stock is UNCHANGED. That is, it still equals s at the end of the second quarter.

Now, plug x = 0 and y = 0 into answer choices and see which one yields s as the OUTPUT.
(A) s(1+x)(1-y) = s works - keep
(B) s(x/100)(y/100) = 0 ELIMINATE
(C) s(1+[x/100])(1-[y/100]) = s works - keep
(D) 100s(x+y) = 0 ELIMINATE
(E) xy(100+[s/100]) = 0 ELIMINATE

Great we're QUICKLY down to 2 options:
(A) s(1+x)(1-y)
(C) s(1+[x/100])(1-[y/100])

Let's try another set of extreme values.
If x and y both equal 100, we have a situation where the stock increases 100% (i.e. DOUBLES) in first quarter and decreases by 100% in second quarter. A decrease of 100% means the stock loses ALL OF ITS VALUE.
So, the stock is is worth 0 at the end of the second quarter.

Now, plug x = 100 and y = 100 into answer choices and see which one yields 0 as the OUTPUT.
(A) s(1+x)(1-y) = some negative value - ELIMINATE
(C) s(1+[x/100])(1-[y/100]) = 0 perfect

Answer: C

Cheers,
Brent
Brent Hanneson - Creator of GMATPrepNow.com
Image

Legendary Member
Posts: 518
Joined: Tue May 12, 2015 8:25 pm
Thanked: 10 times

by nikhilgmat31 » Tue Jul 14, 2015 2:31 am
answer is C